Đến nội dung

le truong son

le truong son

Đăng ký: 24-10-2015
Offline Đăng nhập: 14-07-2018 - 00:03
***--

#710672 Đề thi tuyển sinh vào lớp 10 chuyên toán Đại học Vinh năm học 2018-2019

Gửi bởi le truong son trong 11-06-2018 - 23:57

 

 

Câu 4 (1,0 điểm). Cho các số thực không âm $a,b$ thỏa mãn $(a-b)^2=a+b+2$. Chứng minh rằng $(1+\frac{a^3}{(b+1)^3})(1+\frac{b^3}{(a+1)^3}) \leq 9$.

 

$(a-b)^2=a+b+2=>a^2+b^2+a+b=2(a+1)(b+1)$=>$\frac{a}{b+1}+\frac{b}{a+1}=2$

$A=\left ( 1+\frac{a^3}{(b+1)^{3}} \right )\left ( 1+\frac{b^3}{(b+1)^{3}} \right )=9+\frac{a^3b^3}{(a+1)^{3}(b+1)^{3}}-6\frac{ab}{(a+1)(b+1)}$

Đặt $t=\frac{ab}{(a+1)(b+1)}$ , dễ thấy $t^3-6t=t(t^2-6)\leq 0$ => đpcm




#662646 Chứng minh $\sum\frac{1+x^{2}}{1+y+z...

Gửi bởi le truong son trong 21-11-2016 - 20:48

Mở rộng cho bài 3: 

Cho x, y, z > 0 và x + y + z = 3. Chứng minh:

$\sqrt[3]{x}+\sqrt[3]{y}+\sqrt[3]{z}\geq xy+yz+xz$ 

 Có trong quyển sáng tạo BĐt của PKH, mình xin trình bày lại cách giải:

Áp dụng BĐT holder: $(\sum \sqrt[3]{x})^3(\sum x)^5\geq (\sum x^{\frac{3}{4}})^8$

Đặt $x^\frac{3}{4}=a;y^\frac{3}{4}=b;z^\frac{3}{4}=c$

Cần chứng minh: $(a^3+b^3+c^3)^8\geq 3^5(a^4b^4+b^4c^4+c^4a^4)^3$(1)

Chuẩn hóa: $a^3+b^3+c^3=3$ =>(1)<=>$3\geq \sum a^4b^4(2)$

Ta có:$\sum a^4b^4=\sum a^3b^3.ab\leq \sum a^3b^3.\frac{4-c^3}{3}= \frac{\sum 4a^3b^3-3a^3b^3c^3}{3}$

(2)=>$4\sum a^3c^3-3a^3b^3c^3\leq 9<=>4(\sum a^3b^3)(a^3+b^3+c^3)\leq (a^3+b^3+c^3)^3+9a^3b^3c^3<=>\sum a^9+3a^3b^3c^3\geq \sum a^3b^3(a^3+b^3)$(hnđ)

=>đpcm




#662157 Có tồn tại n thuộc N $n^2+n+1 \vdots 49$ hay không?

Gửi bởi le truong son trong 16-11-2016 - 19:46

Có tồn tại n thuộc N $n^2+n+2 \vdots 49$ hay không?

4A= $(2n+1)^2+7$

4A chia hết 49=> 4A chia hết 49

Ta có : A chia hết 7=>$(2n+1)^2 chia hết 7=>$(2n+1)^2 chia hết 49

=>4A không chia hết 49

=>A không chia hết 49




#662156 Chứng minh rằng: $\frac{a}{1+b+ac}+\frac...

Gửi bởi le truong son trong 16-11-2016 - 19:40

Cho ba số thực $a,b,c\in[0;1]$. Chứng minh rằng:

$\frac{a}{1+b+ac}+\frac{b}{1+c+ab}+\frac{c}{1+a+bc}\leq 1$

Ta có: $(a-1)(b-1)\geq 0=>ab+c+1\geq a+b+c$=>$\sum \frac{a}{1+b+ac}\leq \sum \frac{a}{a+b+c}=1$




#662120 Đề thi chọn HSGQG 2017 Đại học KH Huế

Gửi bởi le truong son trong 16-11-2016 - 11:08

15129774_1809479515995827_869966254_n (1).jpg

 




#661952 cho a,b,c>0.CMR: $\sum \sqrt{\frac{a}...

Gửi bởi le truong son trong 14-11-2016 - 22:12

1.cho a,b,c>0.CMR: $\sum \sqrt{\frac{a}{2a+b+c}}\leq \frac{3}{2}$

2.Cho $\sum \sqrt{a^2+b^2}=\sqrt{2011}$. CMR $\sum \frac{a^2}{b+c}\geq \frac{1}{2}\sqrt{\frac{2011}{2}}$

Cách khác bài 2(khá đẹp):

$\sum \frac{a^2}{b+c}\geq \sqrt{2}\sum \frac{a^2}{\sqrt{b^2+c^2}}$

Đặt $\sqrt{b^2+c^2}=x;\sqrt{c^2+a^2}=y;\sqrt{a^2+b^2}=z=>x+y+z=\sqrt{2011} =>P=\sqrt{2}\sum \frac{y^2+z^2-x^2}{2x}=\sum \frac{y^2+z^2}{\sqrt{2}x}-\sum \frac{x}{\sqrt{2}}\geq 2.\frac{(x+y+z)^2}{\sqrt{2}(x+y+z)}-\frac{x+y+z}{\sqrt{2}}=\frac{1}{2}\sqrt{\frac{2011}{2}}$




#660023 109 bất đẳng thức

Gửi bởi le truong son trong 30-10-2016 - 22:13

Không có ý gì đâu. Mình rất phục bạn khi bạn đổi biến :wub:  nhưng $(x+y-z)(x-y+z)(-x+y+z)\leq xyz$ là bất đẳng thức Shur mà :icon13:  , đừng có đùa mình chứ. Chắc là bạn chứng minh lại hoặc không để ý thôi :lol:

@@,chắc tại mình k để ý:v, lm phức tạp lên




#659993 109 bất đẳng thức

Gửi bởi le truong son trong 30-10-2016 - 19:32

2. Cho $a,b,c$ là những số thực dương thỏa mãn $abc=1$, chứng minh rằng:

$\left ( a-1+\frac{1}{b} \right )\left ( b-1+\frac{1}{c} \right )\left ( c-1+\frac{1}{a} \right )\leq 1$

Đổi biến $(a;b;c)= (\frac{x}{y};\frac{y}{z};\frac{z}{x})$

Bất đẳng thức cần chứng minh <=>$(x+y-z)(x-y+z)(y+z-x)\leq xyz$( chứng minh BĐT này khá khó :( )

Giả sử $x\geq y\geq z$

TH1:$y+z>x$

Ta có: $x+y>z;y+z>x;z+x>y$

=>x;y;z là 3 cạnh của 1 tam giác

Áp dụng phép thế Ravi, đặt $x=p+q;y=p+r;z=q+r$(p,q,r>0)

Khi đó (1)<=>$(y+z)(z+x)(x+y)\geq 8xyz<=>x(y-z)^2+z(x-y)^2+y(x-z)^2$\geq 0$(hnđ)

=>đpcm

TH2:$y+z\leq x=>xyz$> 0$\geq (y+z-x)(x+z-y)(x+y-z)$

=>đpcm

Vậy BĐT được chứng minh :D




#658710 pt: x3-x2+3ax-b=0 có 3 nghiệm.

Gửi bởi le truong son trong 21-10-2016 - 21:12

Bài 1:

Cho :  a,b>0

pt: x3-x2+3ax-b=0 có 3 nghiệm.

CMR: a3/b3 + 27b>=28

Bài 2:

Trong cặp nghiệm của pt : x2-x2y-y+8x+7=0.

Tìm cặp nghiệm (x,y) mà y max.

Bài 1: Gọi m;n;p là 3 nghiệm của Pt=>m,n,p>0

Theo hệ thức Viét:$m+n+p=1;mn+np+pm=3a;mnp=b$

Ta có $mn+np+mp\geq 3\sqrt[3]{m^2n^2p^2}=>3a\geq 3\sqrt[3]{b^2}=>\frac{a^3}{b^3}\geq b$

Cần chứng minh: $\frac{a^3}{b^3}+27b\geq 28$

Thật vậy : $\frac{a^3}{b^3}+27b\geq \frac{1}{b}+27b$

$\frac{1}{b}+27b\geq 28<=>(1-b)(1-27b)\geq 0(1)$

Mặt khác: $1=m+n+p\geq 3\sqrt[3]{b}=>b\leq \frac{1}{27}$

=>(1) Đúng

=>đpcm




#658676 Sử dụng BĐT AM-GM trong chứng minh BĐT.

Gửi bởi le truong son trong 21-10-2016 - 16:46

attachicon.gifCapture.PNG                                                                                                                                                                                                                                                              

  43, Ta có:$\sum \frac{x^2y}{z}\sum \frac{x^2z}{y}\geq (x^2+y^2+z^2)^2$                                                                                                                                             Mặt khác: $\sum \frac{x^2y}{z}-\sum \frac{x^2z}{y}=\frac{(xy+yz+xz)(x-y)(y-z)(x-z)}{xyz}\geq 0$(hnđ vì $a\geq b\geq c$)

44, Từ giả thiết=>$\sum \frac{x-1}{x}=1=>x+y+z=(x+y+z)\sum \frac{x-1}{x}\geq (\sum \sqrt{x-1})^2 =>\sqrt{x+y+z}\geq \sum \sqrt{x-1}$

45,Ta có: $\sum \frac{a}{4b^2+1}=\sum \frac{a^3}{4a^2b^2+a^2}\geq \frac{(a\sqrt{a}+b\sqrt{b}+c\sqrt{c})^2}{\sum 4a^2b^2+\sum a^2}$

Cần chứng minh: $\sum 4a^2b^2+\sum a^2\leq 1=(a+b+c)^2<=>\sum ab(1-2ab)\geq 0(hnđ vì a+b+c=1)$=>đpcm

41,f=$3-\sum \frac{1}{x+1}\leq 3-\frac{9}{x+y+z+3}=3-\frac{9}{4}=\frac{3}{4}$




#658280 $\sqrt{\frac{a^2}{a^2+b+c}}+...

Gửi bởi le truong son trong 18-10-2016 - 12:57

 

      Bài 2: Chứng minh rằng với mọi $a,b,c > 0$, ta có:

$\frac{a^4}{1+a^2b}+\frac{b^4}{1+b^2c}+\frac{c^4}{1+c^2a} \geq \frac{abc(a+b+c)}{1+abc}$

     

Từng gặp 1 bài toán tương tự, không bt có liên quan không: Chứng minh rằng với mọi a,b,c>0, ta có:$\sum \frac{a^3b}{1+ab^2}\geq \frac{abc(a+b+c)}{1+abc}$

Giải như sau: Với mọi số dương k, ta có$\sum \frac{a^2}{b+kc}\geq \frac{(a+b+c)^2}{a+b+c+ka+kb+kc}=\frac{(a+b+c)^2}{(a+b+c)(k+1)}=\frac{a+b+c}{k+1}$

Chọn $k=\frac{1}{abc}$ ta có đpcm :D




#658263 Max: $F=\frac{x^{4}}{(x^{2}+y^...

Gửi bởi le truong son trong 18-10-2016 - 00:23

Cho các số thực dương $x, y, z$ thỏa mãn điều kiện: $x+y+z=1$ . Tìm giá trị nhỏ nhất của biểu thức: $F=\frac{x^{4}}{(x^{2}+y^{2})(x+y)}+\frac{y^{4}}{(y^{2}+z^{2})(y+z)}+ \frac{z^{4}}{(z^{2}+x^{2})(z+x)}$

Dễ thấy:$\sum \frac{x^4}{(x+y)(x^2+y^2)}-\sum \frac{y^4}{(x+y)(x^2+y^2)}=\sum x-\sum y=0$

=>$\sum \frac{x^4}{(x+y)(x^2+y^2)}=\frac{1}{2}(\sum \frac{x^4}{(x+y)(x^2+y^2)}+\sum \frac{y^4}{(x+y)(x^2+y^2)})=\frac{1}{2} \sum \frac{x^4+y^4}{(x+y)(x^2+y^2)}\geq \frac       {1}{4}\sum \frac{(x^2+y^2)^2}{(x+y)(x^2+y^2)}\geq \frac{1}{8}\sum \frac{ (x+y)^2}{x+y}=\frac{1}{8}\sum (x+y)=\frac{1}{4}$

=>đpcm




#657957 $\left | 2c^2+ac+b \right |\geq 1$

Gửi bởi le truong son trong 15-10-2016 - 20:30

1.Chứng minh với mọi a,b luôn tồn tại $c\in \left [ -1;1 \right ]$ sao cho :

$\left | 2c^2+ac+b \right |\geq 1$ 

2.Tìm m max của hàm số $y=\left | x^2-4x+m \right |$ với $x\in \left [ 1;4 \right ]$ đạt min

1,Giả sử không $c\in \left [ -1;1 \right ]$ sao cho :$\left | 2c^2+ac+b \right |\geq 1$ 

=>$\forall c\epsilon[-1;1]=>f(c)=\left |2c^2+ac+b \right |<1$

Ta có: $f(0)<1=>\left | b \right |<1=>1>b>-1$

$f(-1)<1=>\left | 2-a+b \right |<1$

$f(1)=\left | 2+a+b \right |<1$

=>$f(-1)+f(1)<2$

Mặt khác: $f(-1)+f(1)=\left |2-a+b \right |+\left | 2+a+b \right |\geq \left | 4+2b \right |>2$

=>Mâu thuẫn

=>đpcm :D




#657937 Đề thi chọn đội tuyển trường THPT chuyên LQĐ Ninh Thuận khối 10 lần 1, năm họ...

Gửi bởi le truong son trong 15-10-2016 - 19:07

 

 

 

Bài 4: (3,0đ)

          Tìm tất cả các cặp số tự nhiên $(x,y)$ thỏa mãn phương trình $2^x=y^2-135$.

 

 

Xét x lẻ=>x=2k+1=>$2^x=2^{2k+1}=2.4^x=2.(5-1)^x\equiv 2.(-1)^x\equiv 2;3(mod5)$

Mà $y^2-135\equiv 0;1;4(mod5)$

=>x chẵn

=>x=2k=>$2^{2k}-y^2=135<=>(2^k-y)(2^k+y)=135$

Đến đây thì dễ r




#657793 $\sum \sqrt{\frac{a^2+bc}{b^2+bc+c^2...

Gửi bởi le truong son trong 13-10-2016 - 23:17

  1. Cho a,b,c>0 và ab+ac+bc>0 chứng minh: $\sum \sqrt{\frac{a^2+bc}{b^2+bc+c^2}}\geq \sqrt{6}$